PATOLOGÍA TERCER PARCIAL
![]() |
![]() |
![]() |
Título del Test:![]() PATOLOGÍA TERCER PARCIAL Descripción: PATOLOGÍA TERCER PARCIAL |




Comentarios | |
---|---|
| |
| |
FIN DE LA LISTA |
Niño 7 años con Hematuria de 24 horas, con oliguria, HTA, elevación de urea y creatinina. Antecedente de faringoamigdalitis de repetición, y positividad para Anticuerpos anti estreptococo. Usted diagnostica Síndrome nefrítico, se realiza biopsia renal y el patólogo observa proliferación endocapilar (Glomérulo hipercelular con proliferación del mesangio y obliteración de luces capilares además de infiltración por polimorfonucleares y monocitos además de edema intersticial) y la microcopia electrónica revela depósitos densos subepiteliales en forma de jorobas y diagnostica Glomerulonefritis postestreptococcica. Mencione que es lo que observó en la inmunofluorescencia: Depósitos lineales de Inmunoglobulina A en membrana basal Glomerular. Depósitos granulares de Ig M en mesangio. Depósitos granulares de Ig G y C3 en Membrana basal glomerular y mesangio. Paciente masculino de 20 años con Hematuria, oliguria, HTA, elevación de urea y creatinina. Antecedente de faringoamigdalitis de repetición, y positividad para Anticuerpos anti-estreptococo. El paciente presenta un rápido deterioro y cae en insuficiencia renal aguda. La biopsia muestra glomérulos con proliferación endocapilar y la presencia de medias lunas epiteliales en el 70% de ellos, además de depósitos granulares de Ig G en la membrana basal glomerular y mesangio. Señale el diagnostico más probable. Glomerulopatía membranosa. Glomerulopatía post estreptocócica. Glomerulopatía de cambios mínimos. Enfermedad de Berger. Glomerulonefritis rápidamente progresiva. Hombre 35 años con antecedente de cuadro gripal dos semanas antes que inicia con edema generalizado, disnea, hemoptisis aguda y oliguria los exámenes de laboratorio muestran elevación de urea y creatinina en suero, proteinuria y eritrocituria con cilindros hemáticos. El patólogo reporta Síndrome de Goodpasture, se le administran esteroides y mejora rápidamente. Señale contra que elemento está dirigida la respuesta inmune de este paciente: Contra el mesangio. Contra la membrana basal glomerular y alveolar. Contra el epitelio visceral. Contra el epitelio parietal. Femenina de 32 años con diagnóstico de Lupus Eritematoso Sistémico y daño renal crónico que desarrolla súbitamente disnea, oliguria, hematuria e hipertensión arterial, a la exploración muestra rash en región malar, edema de articulaciones. La imagen de tórax muestra derrame pleural bilateral la biopsia renal muestra glomérulos hipercelulares con colapso de luces capilares, membrana basal engrosada de aspecto de asas de alambre así como cuerpos hematoxilinicos y necrosis fibrinoide, en 70 % de los glomérulos. Mencione el diagnostico más probable. Glomerulopatía Lúpica Clase III. Glomerulopatía Lúpica Clase IV. Glomerulopatía Lúpica Clase V. Glomerulopatía rápidamente progresiva. Niño de 4 años, con edema palpebral de 10 días de evolución, los exámenes de laboratorio muestras proteinuria selectiva (solo albúmina), hipoalbuminemia, hiperlipidemia, y lipiduria. El patólogo diagnostica Enfermedad de cambios mínimos, señale cual de los siguientes hallazgos es el necesario para establecer este diagnostico. Medias lunas epiteliales. Cambios proliferativos en la microscopia de luz. Depósitos densos subepiteliales en forma de jorobas. Borramiento de los procesos podálicos. Hombre de 28 años con hepatitis C presenta orina oscura, de dos semanas de evolución, a la exploración presenta hipertensión. Los exámenes de laboratorio muestran elevación de creatinina sérica (8 3.8mg/dl) y del nitrógeno ureico (35mg/dl) se detectan crioglobulinas. El examen de orina muestra cilindros de eritrocitos. La biopsia renal muestra glomérulos hipercelulares lobulados con membranas basales con imagen de doble contorno de aspecto de vías de tren en la tinción de plata separada y depósitos de inmunocomplejos subendoteliales. Señale que células proliferan en estos glomérulos. Células mesangiales. Células epiteliales parietales. Podocitos. Células endoteliales. Un estudio de enfermedades renales identifica a pacientes con más de 3.5g de proteínas en las muestras de orina de 24 horas, ¿Cuál disfunción celular produce esta proteinuria (MASIVA)?. Endotelio. Macula densa. Mesangio. Epitelio parietal. Podocitos. Masculino con SIDA en tratamiento, con edema generalizado, las pruebas de laboratorio muestran proteinuria no selectiva, y hematuria microscópica súbitamente desarrolla descenso en el filtrado glomerular la biopsia renal muestra afectación glomerular focal, y algunas partes de los glomérulos muestra retracción y colapso con proliferación e hipertrofia del epitelio visceral, con marcadas áreas de adherencias a la cápsula (colapso). Algunos túbulos presentan micro quistes llenos de material proteináceo con inflamación y fibrosis. Señale el diagnóstico y mas probable en este paciente según sus antecedentes. Glomeruloesclerosis focal y segmentaria clásica. Glomeruloesclerosis focal y segmentaria variedad colapsante. Hombre 28 años. Antecedente de endocarditis. Desarrolla proteinuria, hematuria e hipertensión leve. Se realiza biopsia renal que muestra proliferación de matriz mesangial, de células mesangiales y del endotelio con colapso de luces capilares e infiltración por leucocitos polimorfonucleares. La tinción de plata muestra desdoblamiento de la membrana basal en vías de tren. La inmunofluorescencia muestra depósitos granulares de inmunoglobulinas y complemento C3 en capilares y mesangio. La microscopia electrónica muestra depósitos subendoteliales. Señale el diagnóstico más probable: Glomerulonefritis membrana proliferativa (antes Tipo I). Enfermedad por depósitos densos (antes tipo II). Glomeruloesclerosis focal y segmentaria. Hombre de 44 años con edema generalizado, disnea y ascitis. Presenta proteínas séricas totales es de 5,2 g / dL (referencia = 5,5 a 8,0 g / dL) y la albúmina es 1,9 g / dL, (referencia = 3,5 a 5,5 g / dL). El colesterol sérico se eleva a 530 mg / dL. Hay 5 g de proteína en una colección de orina de 24 horas. El sedimento urinario contiene muchos cilindros de hialina pero no eritrocitos, ni leucocitos. La biopsia renal muestra engrosamiento de la membrana basal glomerular con aspecto de espigas que corresponde a depósitos granulares de Ig G ¿Cuál de los siguientes es el diagnóstico más probable?. Glomerulopatía lúpica Clase IV. Glomerulopatía lúpica Clase V. Glomerulopatía rápidamente progresiva. Glomerulopatía Membranosa. Niño de 12 años con anemia hemolítica en estudio, desarrolla dolor en hipocondrio derecho, que se irradia en hemicinturón hacia la escapula, se le realiza ecografía abdominal y se encuentra una vesícula aumentada de tamaño con múltiples cálculos facetados. Según los antecedentes de este paciente de que color serían los cálculos: Amarillos. Negros. Pardos. Femenina de 52 años con ictericia y dolor en hipocondrio derecho que se irradia en hemicinturón a escapula derecha, fiebre, ataque al estado general ictericia. En la ecografía observan una vesícula con lesión polipoide e irregular en el fondo, cálculos y engrosamiento de la pared e infiltración al hígado. Con estos datos cuál sería el diagnostico más probable?. Adenoma. Carcinoma. Masculino 50 años con ictericia y pérdida de peso, elevación de bilirrubinas y fosfatasa acida, presenta una lesión en la unión de ambos conductos hepáticos mayores, se toma biopsia y el patólogo diagnostica un carcinoma, como se llama esta neoplasia por estar localizada en estos conductos?. Tumor de Wartin. Tumor de Krukenberg. Tumor de Klatskin. Masculino con pérdida de peso no cuantificada antecedentes de quistes del colédoco inicia con ictericia, fiebre y dolor intermitente El ECO: muestra dilatación de vía biliar, y nódulos correspondientes a ganglios peri duodenales aumentados de tamaño y un tumor polipoide en el ámpula de Vater. ¿Cuál sería el diagnostico más probable?. Adenoma. Carcinoma. Masculino ingresa a urgencias con aliento alcohólico, dolor en epigastrio intenso, hipotensión, ataque al estado general, nausea y vomito. La exploración física muestra signo de Cullen (equimosis periumbilicales) y signo de Grey-Turner (equimosis en ángulo costo lumbar), usted sospecha Pancreatitis aguda, ¿Que enzima considerada especifica de este padecimiento se eleva después de 72 horas?. Amilasa. Fosfatasa alcalina. Lipasa. Tripsina. Masculino de 30 años, sin antecedentes relevantes, con ictericia, en la ecografía presenta un tumor único hepático con zonas de aspecto fibroso en un hígado sin cirrosis. La biopsia muestra un tumor compuesto de hepatocitos que se distribuyen en cordones rodeados de bandas de colágena densa, además de cilindros de bilis. Mencione el diagnóstico más probable. Adenocarcinoma hepático clásico. Adenocarcinoma fibrolamelar. Colangiocarcinoma. Femenina con historia de dolor en hipocondrio derecho que se irradia al hombro en hemicinturón refiere e ictericia con remisiones y exacerbaciones en los últimos dos años acude a consulta por esteatorrea e indigestión, en la imagen presenta una vesícula con litos y edema de la pared, a nivel del páncreas se observa edema y un cálculo impactado en uno de los ductos. ¿Cuál sería el diagnóstico más probable?. Pancreatitis aguda. Pancreatitis crónica. Pseudoquiste pancreático. Absceso pancreático. Masculino de 50 años con tabaquismo positivo y pérdida de peso, anorexia y malestar general, ingresa por dolor intenso en epigastrio a la exploración se observa, enrojecimiento de la pierna derecha en la región pre tibial, la ecografía de la pierna muestra tromboflebitis, el paciente refiere que ese enrojecimiento lo presentó en la otra extremidad un mes antes y se restableció sin secuelas. El TAC de abdomen muestran un páncreas de aspecto fibroso con calcificaciones. Usted sospecha carcinoma del páncreas. ¿Qué marcador tumoral es el más relacionado a esta neoplasia?. Ca 125. CA 19-9. AFP. Masculino de 7 años con vomito, deshidratación y dolor abdominal, ingresa al urgencias con alteraciones de la conciencia y respiración de Kussmaul (respiración rápida, profunda, irregular) y aliento cetónico. Los exámenes de laboratorio muestran cetonemia, cetonuria. Mencione el diagnóstico más probable y la complicación que presenta el paciente: Coma hiperosmolar - diabetes tipo II. Cetoacidosis - Diabetes tipo I. Hipoglicemia - Insulinoma. Femenina de 65 años con ictericia y prurito intenso, datos de malestar intestinal, a la exploración presenta lesiones amarillentas en la piel palpebral, los exámenes de laboratorio muestran elevación de la bilirrubina, fosfatasa alcalina, Gamma glutamiltrasferasa y colesterol, la biopsia hepática muestra destrucción de los conductos hepáticos infiltrado linfoplasmocítico algunos granulomas alternando con zonas sanas, áreas de fibrosis y colestasis peri portal, además de degeneración plumosa de hepatocitos y algunos cuerpos de Mallory, usted diagnostica Colangitis Biliar Primaria. Mencione el estudio que pediría en este caso: Anticuerpos antinucleares (ANCA). Anticuerpos antimitocondriales. Femenina de 40 años con antecedente de traumatismo craneoencefálico dos meses antes, presenta poliuria y polidipsia, glucosa normal. Usted sospecha un origen hipofisiario, señale la etiología más probable: Secreción inapropiada de hormona antidiurética. Deficiencia de hormona antidiurética. Masculino de 60 años con tabaquismo positivo desde los 14 años y diagnóstico de Enfermedad pulmonar obstructiva crónica, tres meses antes del ingreso presenta debilidad y letargia que progresa al coma, se le diagnostica Edema cerebral, y en los exámenes de Laboratorio se observa Hiponatremia, la tele de tórax muestra un nódulo pulmonar. Señale el diagnóstico más probable: Secreción inapropiada de hormona antidiurética. Deficiencia de hormona antidiurética. Femenina con embarazo a término presenta ruptura uterina por acretismo placentario con hipovolemia, se le transfunde y sobrevive. Desarrolla agalactia y posteriormente falta de menstruación, con el paso del tiempo se agrega palidez cutánea, pelo seco y quebradizo. Señale cual es la causa mas probable de ese desequilibrio hormonal: Síndrome de Sheehan. Apoplejía hipofisiaria. Adenoma hipofisiario. Femenina con antecedente de hiperprolactinemia, tratado con bromocriptina de 6 meses de evolución, que presenta hemianopsia bitemporal de una semana de evolución, desarrolla cefalea súbita, diplopía y perdida de la conciencia, a la exploración presenta edema de la papila y se le diagnostica hipertensión intracraneal. Señale el diagnostico más probable: Síndrome de Sheehan. Apoplejía hipofisiaria. Adenoma hipofisiario. Síndrome de la silla turca vacía. Masculino sin antecedentes médicos relevantes con traumatismo craneoencefálico en accidente vial, se le realiza una imagen de cráneo y en la imagen se observa una silla turca ensanchada con área hipodensa en la zona selar. Mencione el diagnóstico más probable: Síndrome de Sheehan. Apoplejía hipofisiaria. Adenoma hipofisiario. Síndrome de la silla turca vacía. Varón de 14 años sin desarrollo puberal, a la exploración se detecta anosmia. Señale el síndrome más probable. Síndrome de Laron. Síndrome de Kallman. Síndrome de Sheehan. Síndrome de Nelson. Femenina 42 años oligomenorrea, obesidad central, cara de luna llena, estrías abdominales y jiba en cuello. Antecedente de Hiperplasia bilateral suprarrenal con resección de ambas suprarrenales. Desarrolla cefalea y alteraciones visuales e hipertensión intracraneal, la imagen radiologica muestra un macroadenoma hipofisiario. Señale el diagnóstico más probable: Síndrome de Laron. Síndrome de Kallman. Síndrome de Sheehan. Síndrome de Nelson. Niño de 8 años con Cefalea, Defectos visuales se encuentra disminución de las hormonas hipofisiarias. La imagen muestra un tumor en la región selar solido quístico con calcificaciones. Se realiza la resección y el neurocirujano refiere que drena liquido semejante al aceite de motor quemado, el patólogo observa una proliferación de tipo epitelial de aspecto escamoso benignos. Señale el diagnóstico más probable: Quiste de Ranke. Craneofaringioma. Adenoma hipofisiario. Mujer 25 años con pérdida de peso de 15 kilos en seis meses taquicardia, sudoración, ansiedad, temblor fino. Los exámenes de Laboratorio muestran TSH alta. Señale el diagnóstico mas probable: Hipertiroidismo primario. Hipertiroidismo secundario. Síndrome paraneoplásico. Bebe de 1 año producto de primera gesta sin control prenatal con talla bajan rasgos faciales toscos y protrusión de la lengua, a la exploración presenta hernia umbilical, y retraso sicomotor, los exámenes muestran Anemia y TSH baja. Señale el diagnostico más probable: Síndrome de Laron. Síndrome de Kallman. Síndrome de Sheehan. Síndrome de Nelson. Cretinismo. Mujer de 40 años con historia de infección respiratoria viral dos semanas después presenta dolor tiroideo, a la exploración se observa tiroides aumentada de tamaño, los exámenes de laboratorio muestran elevación de T3 y T4, disminución de la TSH, la gammagrafía muestra baja captación de Yodo. Al mes presenta recuperación casi completa. ¿Cuál sería el diagnóstico más probable?. Hiperplasia de tiroides. Tiroiditis de Quervain. Tiroiditis de Riedel. Tiroiditis de Hashimoto. Femenina de 48 años con aumento del tamaño de cuello de manera difusa, disnea y disfagia progresiva, rápido crecimiento, a la palpación es de consistencia pétrea o leñosa e irregular la imagen muestra gran masa dependiente del tiroides que infiltra las estructuras de cuello rodando esófago y tráquea. La biopsia muestra tejido tiroideo con infiltración por linfocitos y abundante fibrosis y depósito de colágena. Además presenta anticuerpos antitiroideos. Señale el diagnostico más probable. Hiperplasia de tiroides. Tiroiditis de Quervain. Tiroiditis de Riedel. Tiroiditis de Hashimoto. Mujer 30 años con aumento del tamaño del tiroides, exoftalmos y mixedema pretibial, y elevación de las hormonas tiroideas, usted diagnostica enfermedad de Graves, señale que observaría en la gammagrafía?. Aumento de la captación de yodo de forma difusa. Disminución de la captación de Yodo de manera difusa. No hay captación de yodo. Mujer de 40 años con aumento de peso de 5kg en un año, piel y mucosas secas, pelo quebradizo, habla lenta, en los exámenes de laboratorio presenta elevación de T3 y T4 bajas, y anticuerpos anti-tiroglobulinas. La ecografía muestra una glándula tiroides disminuida de tamaño, se realiza una biopsia y el patólogo diagnostica Tiroiditis de Hashimoto. Mencione que es lo que el patólogo observó en la biopsia. Un tejido tiroideo compuesto de folículos con marcada hiperplasia con la formación de seudopapilas. Tejido tiroideo con inflamación crónica incluyendo la destrucción de folículos con fibrosis y reacción granulomatosa de tipo cuerpo extraño. Tejido tiroideo con infiltrado linfocitico con folículos linfoides, cambios oncocíticos y atrofia. Mujer con nódulo tiroideo, de 2cm de lóbulo derecho del tiroides totaalmente encapsulado, hipercaptante en Gagammagrafía, presenta elevación de T3 y T4, según las características del tumor señale el diagnóstico más probable. Adenoma folicular. Carcinoma folicular. Carcinoma papilar. Mujer con nódulo tiroideo de 2cm en la ecografía se reporta un nódulo con una cápsula y áreas de degeneración quística con calcificaciones se realiza una Biopsia con aspiración con aguja fina (BAAF), y el patólogo reporta una proliferación neoplásica maligna compuesta por células cubicas cuyos núcleos presentan seudoinclusiones y barras nucleares, señale la neoplasia tiroidea que se caracteriza por estos hallazgos. Carcinoma folicular. Carcinoma papilar. Carcinoma anaplásico. Masculino de 22 años con tumor en cuello dependiente del tiroides se le realiza resección y se observa una proliferación de células que se disponen en trabéculas y nidos, con un fondo de material de aspecto amiloide se realiza una tinción de rojo Congo y tiene birrefringencia de color verde manzana, con lo que realiza el diagnóstico de Carcinoma Medular. Después de estudiar al paciente, se le diagnosticó Síndrome de Neoplasia endocrina múltiple tipo 2, señale los otros dos padecimientos que componen este síndrome. Hipertiroidismo y feocromocitoma. Adenoma hipofisiario y tumores pancreáticos. Adenoma paratiroideo y carcinoma suprarrenal. Masculino de 55 años con un tumor en cuello de crecimiento rápido, en la imagen se observa un tumor dependiente del tiroides que infiltra los tejidos blandos del cuello, se realiza una Biopsia y se reporta como una proliferación de células neoplásicas grandes pleomórficas con atipia y marcada anaplasia además de zonas con necrosis. El patólogo reporta un carcinoma tiroideo; con estos datos histológicos señale la variada mas probable de carcinoma de tiroides en este paciente: Carcinoma folicular. Carcinoma papilar. Carcinoma anaplásico. Masculino 22 años con gastritis, estreñimiento y dolor óseo intenso de 1 año de evolución, ingresa a urgencias por hematuria y cólico renal. A la exploración presenta Hipertensión. Los exámenes de laboratorio muestran hipercalcemia y elevación de la hormona paratiroidea. El eco de abdomen muestra múltiples cálculos renales. La imagen de cuello muestra una tiroides normal y entre ella se observa una de las glándulas paratiroideas aumenta de tamaño de 2cm sin grasa intersticial y el resto se ven normales. Señale el diagnostico más probable: Hiperplasia de las glándulas paratiroides. Neoplasia paratiroidea. Femenina con tratamiento crónico con esteroides, que presenta amenorrea, obesidad central, cara de luna llena, estrías abdominales e hipertensión, usted diagnostica Síndrome de Cushing, señale que es lo que espera encontrar en las suprarrenales. Hiperplasia difusa en ambas glándulas. Hiperplasia nodular en ambas glándulas. Atrofia bilateral. Mujer 35 años con tumor hipofisiario que produce ACTH, que espera encontrar en la glándula suprarrenal?. Hiperplasia difusa en ambas glándulas. Hiperplasia nodular en ambas glándulas. Atrofia bilateral. Recién nacido femenino con genitales ambiguos y virilización progresiva, se determina origen suprarrenal, mencione la causa más frecuente de este síndrome de origen congénito: Deficiencia de 21-hidroxilasa. Deficiencia de 11 hidroxilasa. A 48-year-old man presents with recurrent headaches and arthritic pain in his knees of 9 months in duration. He notes that his hat size has recently increased. He also states that he suffers from erectile dysfunction. His past medical history is signifi cant for kidney stones 2 years ago. Physical examination reveals a blood pressure of 170/100 mm Hg. The patient is observed to have coarse facial features and a goiter. Urinalysis reveals glucosuria and hypercalciuria. Which of the following is the most likely explanation for this patient’s clinical presentation?. Excess growth hormone secretion. Excess parathyroid hormone secretion. Excess prolactin secretion. A 34-year-old man complains of sudden attacks of dizziness, blurred vision, and excruciating headaches of 4 months in duration. During one of these attacks, his blood pressure was 180/120 mm Hg. The patient’s father had been treated for thyroid cancer about 15 years ago. Laboratory studies show normal serum levels of aldosterone, renin, and angiotensin. A 24-hour urinalysis reveals increased metanephrines. Episodic hypertension in this patient is most likely caused by a tumor in which of the following endocrine organs?. Kidney. Parathyroid. Adrenal. Pituitary. Producto de término, fallece al nacer, la autopsia muestra implantación baja de orejas, retrocnatia, extremidades dobladas con dislocación de cadera. hipoplasia pulmonar, riñones aumentados de tamaño con múltiples quistes. Las secciones de riñón muestran múltiples quistes originados de los túbulos colectores sin daño glomerular Tipo I según la clasificación de Potter. Se diagnostica Riñón poliquístico del Recién Nacido, señale que otras alteraciones se asocian a esta patología. Quistes hepáticos. Cirrosis hepática. Pareja que acude a consejo genético después de la perdida de su primer hijo con diagnostico de Riñón Poliquístico del recién nacido, señale el riesgo de recurrencia de este síndrome en los hijos. Riesgo de recurrencia en 25% en hijos subsecuentes. Riesgo de recurrencia en 50% en hijos subsecuentes. Óbito (muerte fetal), con oligohidramnios, crecimiento renal bilateral, el riñón muestra quistes irregulares revestidos de epitelio plano y túbulos colectores inmaduros rodeados de estroma laxo, nidos de cartílago primitivo además de abundante mesénquima inmaduro. Señale el diagnostico más probable: Riñón en esponja medular. Riñón poliquístico del recién nacido. Displasia renal multiquística. Hombre de 35 años con hematuria, a la exploración presenta dolor y masa palpable en ambas fosas renales, el ECO muestra múltiples quistes en ambos riñones. Tiene historia de padre fallecido por enfermedad renal no especifica se sospecha Riñón poliquístico del adulto, señale lo que observaría el patólogo en el parénquima renal. Múltiples quistes tubulares característicos de gigantismo tubular (Potter tipo I). Quistes glomerulares que involucran la capsula de Bowman entremezclado con parénquima normal. Quistes revestidos de epitelio plano, túbulos colectores inmaduros, mesénquima y cartílago primitivo. Paciente con Diagnostico de Riñón Poliquístico de Adulto. Mencione el tipo de herencia asociada a esta patología: Autosómica dominante. Autosómica recesiva. Masculino de 25 años con nistagmo y ceguera del ojo derecho e HTA, se agrega hematuria. Los estudios de imagen muestran un tumor en cerebelo de aspecto vascular y un tumor en riñón derecho de 5cm además de quistes en riñón izquierdo, y un tumor en médula suprarrenal. Se realiza nefrectomía radical y el patólogo reporta un carcinoma de células claras. ¿Cuál es el síndrome más probable que presenta este paciente?. Esclerosis tuberosa. Von Hippel Lindau. Mujer 20 años con fiebre, disuria, polaquiuria, tenesmo y dolor en el flanco el EGO muestra leucocituria (Piuria) con Cilindros y más de 100 mil bacterias/ml. Usted diagnostica pielonefritis aguda, señale cual es la vía más frecuente de llegada de las bacterias al riñón. Vía hematógena. Vía ascendente. Mujer de 60 años con Diabetes tipo II, con infecciones urinarias de repetición. El ECO muestra Riñón con atrofia cortical y cicatrices. Se reseca el riñón y al micro tiene esclerosis glomerular, tiroidizacion, fibrosis intersticial e inflamación crónica. ¿Cuál sería el diagnostico en esta paciente?. Pielonefritis aguda. Pielonefritis crónica. Masculino 39 años con hematuria, disuria, fiebre y dolor cólico en fosa iliaca derecha. EGO: sedimento con cristales, leucocitos, eritrocitos, y cilindros. La imagen muestra un calculo de aspecto coraliforme en el sistema pielocaliceal, se reseca el riñón y el patólogo reporta un pielonefritis xantogranulomatosa, ¿Qué tipo de infección se asocia a este hallazgo?. Infección por bacterias coliformes. Infección por Proteus. Infección por parásitos. Paciente masculino de 20 años con retraso mental y crisis convulsivas desde la infancia. Inicia con hematuria y dolor en el franco. La exploración muestra lesiones nodulares cutáneas en región nasal y malar bilateral. La imagen de abdomen muestra el riñón derecho con tumor en la parte media con imagen consistente con grasa y musculo, se reseca y se diagnostica Angiomiolipoma, señale el síndrome mas probable que padece el paciente. Síndrome de Von Hippel Lindau. Síndrome de Esclerosis tuberosa. Masculino de 38 años con dolor en el flanco, acude a consulta al observar coágulos en la orina. El examen de laboratorio muestra elevación de nitrógeno en la urea y creatinina, además de proteinuria y hematuria la ecografía muestra ambos riñones aumentados de tamaño con quistes, se le diagnostica riñón poliquístico del adulto, con lo anterior señale cual otra anormalidad se asocia a este padecimiento: Carcinoma vesical transicional. Quistes hepáticos. Riñón en herradura. Hombre de 35 años, con antecedentes de promiscuidad sexual. Disuria, polaquiuria, Fiebre y leucocitosis. Al tacto rectal la Próstata está crecida a y dolorosa. Hay leucocituria pero el cultivo de secreción prostática es negativo para bacterias, ¿Cuál sería el agente etiológico más probable?. E. Coli. Estreptococo. Clamidia. Hombre 69 años, refiere Infecciones urinarias de repetición. Inicia 2 días antes con disuria leve a moderada, y que se agudiza con obstrucción urinaria, se le realiza cateterismo sin mejoría y se decide realizar un RTU (resección transuretral). El patólogo describe: proliferación del tejido glandular y del estroma fibromuscular además de infiltrado inflamatorio crónico de tipo linfo-histiocitario. ¿Cuál sería el diagnóstico mas probable?. Adenocarcinoma de próstata. Prostatitis aguda. Hiperplasia de la próstata y prostatitis crónica. Prostatitis aguda por E. Coli. Hombre de 65 años no fumador, sufre una caída desde su propia altura, a la revisión en urgencias presenta una fractura de la cadera derecha, la cual se considera fractura patológica. La imagen revela varias lesiones redondeadas blásticas en el fémur, se sospecha una neoplasia metastásica en el hueso, por el sexo y la edad de este paciente, mencione que estudios le pediría para buscar el origen del tumor primario más probable en este paciente: Ecografía hepática y determinación de bilirrubina. Imagen de pulmón y APE. Masculino de 65 años con aumento de volumen de ambos testículos. LAB: PLAP y LDH negativas. Se realiza resección de ambos testículos observando túbulos seminíferos atróficos con disminución de la espermatogénesis, por fuera de los túbulos se observa infiltración de células de escaso citoplasma y núcleo hipercromático, con escaso citoplasma que infiltran en forma de monocapa. ¿Cuál es el diagnóstico más probable?. Seminoma clásico. Infiltración por linfoma. Tumor espermatocítico. Tumor germinal mixto. Masculino de 23 años. Con aumento del volumen del testículo izquierdo. A la exploración física presenta un testículo aumentado de tamaño, no doloroso. ECO: tumor único, heterogéneo, con necrosis, de bordes mal definidos que infiltra el tejido adyacente. LAB: AFP, PLAP y LDH elevadas. Se programa al paciente para la resección quirúrgica. ¿Cuál es el diagnóstico más probable?. Seminoma clásico. Infiltración por linfoma. Tumor espermatocítico. Tumor germinal mixto. Masculino de 29 años con tumor testicular ECO: tumor homogéneo solido con algunas zonas necróticas bordes redondeados. El patólogo reporta una neoplasia compuesta de células poliédricas con citoplasma claro (con glucógeno) entre las cuales se observan algunos tabiques fibrosos con linfocitos. ¿Cuál sería el diagnóstico más probable en este paciente?. Seminoma clásico. Infiltración por linfoma. Tumor espermatocítico. Tumor germinal mixto. Niño de 1 año 8 meses con aumento de volumen testicular desde el nacimiento. ECO: tumor testicular sólido y quístico con áreas de aspecto óseo. AFP, PLAP y LDH negativas. Señale el diagnóstico más probable. Tumor de senos endodérmicos. Teratoma. Infiltración por linfoma. Niño de 1 año con masa bien delimitada de 2,5 cm en el testículo izquierdo. Se realiza una orquiectomía izquierda: el patólogo reporta una neoformación compuesta de células cúbicas, algunas de las cuales contienen glóbulos hialinos eosinófilos que se disponen en láminas de células, micro quistes y estructuras glomeruloides primitivas. Laboratorio: elevación de alfa feto proteína (AFP), ¿Cuál es el diagnóstico más probable?. Tumor de senos endodérmicos. Teratoma maduro. Infiltración por linfoma. Niño de 6 años con pubertad precoz. El ECO testicular muestra un nódulo heterogéneo de 1.9cm en testículo izquierdo. Se reseca el nódulo el cual es de color amarillo marrón, las secciones muestran células con abundante citoplasma además de observarse cristales de Reinke. Estas células muestran positividad para inhibina. Señale el diagnostico más probable. Tumor de senos endodérmicos. Teratoma maduro. Infiltración por linfoma. Tumor de células de Leydig. Niño de 2 años con una masa en testículo derecho. Se realiza orquiectomía, el examen microscópico de la pieza quirúrgica muestra células neoplásicas que forman estructuras glomeruloides (cuerpos de Schiller-Duval), ¿Cuál de los siguientes marcadores séricos es más útil para monitorear la recurrencia del tumor en este paciente?. CA-125. ACE: Antígeno carcinoembrionario. Estrógeno. AFP (Alfafetoproteína). HGC (Hormona Gonadotropina coriónica humana). Masculino de 25 años con dificultad respiratoria de una semana de evolución, se agrega ictericia y posteriormente desarrolla cefalea, con deterioro súbito y perdida de la conciencia. Se realiza exploración física. Los estudios de imagen muestran 10 nódulos de distintos tamaños (el mayor de 2cm y el menor de 0.5cm en la unión de la substancia blanca y la gris en ambos hemisferios cerebrales, la imagen pulmonar muestra múltiples nódulos de 1 a 2cm de diámetro en ambos pulmones, el Eco de hígado mostró 6 lesiones nodulares la mayor de 5cm y la menor de 2cm. Usted sospecha de lesiones metastásicas múltiples, señale el origen y el tipo histológico mas probable del tumor primario según la edad y sexo del paciente. Seleccione la respuesta correcta: Tumor prostático – adenocarcinoma. Tumor pulmonar – carcinoma indiferenciado de células pequeñas. Tumor testicular – tumor germinal Mixto con coriocarcinoma. Masculino de 35 años. Acude a consulta por presentar dolor agudo en hipocondrio derecho de 2 días de evolución. Dentro de sus antecedentes refiere diagnóstico de anemia hemolítica desde hace 5 meses. Los estudios de imagen muestran múltiples litos en la vesícula biliar. De acuerdo al cuadro clínico anterior indique el tipo de litos que esperaría encontrar en esta paciente: Litos amarillos. Litos pardos. Litos negros. Masculino de 70 años con presencia de pérdida de peso importante además de presentar hiperbilirrubinemia y discreta cianosis en miembros inferiores que al decir del paciente era “cambiante” unos días en la extremidad inferior derecha y a los 5-6 días esta coloración se presentaba en la extremidad inferior contralateral y viceversa. En la exploración física se encontró vesícula biliar palpable no dolorosa además de dilatación de vías biliares extra-hepáticas en el ultrasonido.Con estos datos clínicos indique ¿Cuál es el diagnóstico más probable?. Pancretitis aguda. Síndrome de Budd-Chiari. Síndrome de Boerhave. Cáncer de cabeza de páncreas. Cirrosis hepática por virus hepatotropos. Femenina de 34 años. Acude a consulta por presentar pérdida de peso, cefalea intensa, temblor e intolerancia al calor. A la exploración física presenta un aumento de volumen de la cara anterior del cuello además de alteraciones en la visión. Los estudios de laboratorio reportan elevación de T3, T4 y TSH.De acuerdo al cuadro clínico anterior indique el diagnóstico más probable: Enfermedad de Graves. Tiroiditis de Hashimoto. Adenoma tiroideo. Adenoma hipofisiario. Tiroiditis de Riedel. Mujer de 28 años. Acude al servicio de urgencias por presentar taquicardia, sudoración, ansiedad, temblor fino y refiere una perdida de peso de 10 kg en 2 meses. Los estudios de laboratorio reportan una elevación sérica de T3 yT4, así como una elevación de TSH. De acuerdo al cuadro clínico anterior y pruebas de laboratorio indique cual es su posibilidad diagnostica: Hipotiroidismo primario. Hipotiroidismo secundario. Hipertroidismo primario. Hipertiroidismo secundario. Mujer de 28 años. Acude al servicio de urgencias por presentar taquicardia, sudoración, ansiedad, temblor fino y refiere una perdida de peso de 10 kg en 2 meses. Los estudios de laboratorio reportan una elevación sérica de T3 yT4, así como una elevación de TSH. De acuerdo a su diagnóstico anterior señale la causa más común de esta patología: Adenoma troideo. Adenoma paratiroideo. Adenoma corticosuprartenal. Adenoma hipofisiario. Paciente masculino de 12 años que acude por datos de hipertensión craneal y alteraciones del campo visual caracterizados por una hemianopsia bitemporal. Los estudios de imagen muestran una lesión supraselar por lo que se decide llevar a quirófano, durante el transoperatorio el neurocirujano retiero que la lesión es sólida con degeneración quistica drenando un liquido en aspecto de aceite de motor quemado.¿Cuál es su diagnóstico?. Adenoma hipofisiario funcionante. Adenoma hipofisiario no funcionante. Graneofaringioma. Teratoma. Astrocitoma piocitico. Femenina de 30 años. Acude a consulta de Endocrinología por presentar aumento de volumen de la cara anterior del cuello. A la exploración física presenta exoftalmos y mixedema pretibial. Los estudios de laboratorio reportan una elevación de T3 y T4. Indique de los abajo enlistados, los anticuerpos que esperaría encontrar en esta paciente. Anticuerpos antitiroglobulina. Anticuerpos antiperoxidasa. Anticuerpos antimitocondriales. Anticuerpos contra el receptor de TSH. Anticuerpos antinucleares. Femenina de 30 años Acude a consulta por presentar aumento de peso de 5 kg en 3 semanas. A la exploración física presenta resequedad de piel y mucosas, pelo quebradizo, además de habla lenta. Los estudios de laboratorio reportan una disminución de T3 y T4 en sangre periférica. Los estudios de imagen muestran una glándula tiroides disminuida de tamaño. Se realza cirugia, el estudio histopatológico reporta un infiltrado inflamatorio a base de linfocitos que se disponen formando folículos linfoides, además de cambios oncociticos y foliculos con atrofia. De acuerdo al cuadro clinico anterior indique el diagnóstico más probable: Bocio por deficiencia de Yodo. Enfermedad de Graves Basedow. Tiroiditis Crónica de Hashimoto. Tiroiditis de Quervain. Hombre de 40 años con debilidad y fatiga en los últimos dos meses. En la exploración física no se encuentran alteraciones. Las pruebas de laboratorio muestran elevación del calcio y de la hormona paratiroidea. Los estudios radiográficos mostraron una osteítis fibrosa quística. ¿Cuál es la causa más probable de estos hallazgos?. Hiperplasia de paratiroides. Adenoma paratiroideo. Carcinoma medular de tiroides. Carcinoma paratiroideo. Hipervitaminosis D. Masculino de 25 años de edad. Acude a consulta por presentar múltiples cálculos renales, poliuria, hipertensión además de estreñimiento crónico. Al interrogatorio intencionado refiere dolor óseo intenso. Se realizan estudios de laboratorio que reportan hipercalcemia. Además los niveles de hormona paratiroidea elevados. La fosfatasa alcalina se encuentre en 190UE/L. Por lo que se decide realizar estudios de imagen que reportan la presencia de glándula paratiroidea inferior aumentada de tamaño con un peso aparente por estudio de imagen de 0/7gr. El resto de las glándulas paratroideas se encuentran dentro de limites normales. En este caso clínico cual seria su diagnóstico más probable: Hiperplasia de paratiroides. Adenoma de paratiroides. Carcinoma de paratiroides. Masculino de 25 años de edad. Acude a consulta por presentar múltiples cálculos renales, poliuria, hipertensión además de estreñimiento crónico. Al interrogatorio intencionado refiere dolor óseo intenso. Se realizan estudios de laboratorio que reportan hipercalcemia. Además los niveles de hormona paratiroidea elevados. La fosfatasa alcalina se encuentre en 190UE/L. Por lo que se decide realizar estudios de imagen que reportan la presencia de glándula paratiroidea inferior aumentada de tamaño con un peso aparente por estudio de imagen de 0/7gr. El resto de las glándulas paratroideas se encuentran dentro de limites normales. En base a los hallazgos clínicos y de laboratorio encontrados por usted en este paciente se consideraría a esta patología y cuadro clínico como un ejemplo clásico de: Hipoparatiroidismo primario. Hiperparatiroidismo secundario. Hiperparatiroidismo terciario. Hiperparatiroidismo primario. Hipoparatiroidismo secundario. Paciente masculino de 30 años con hipertensión arterial refractaria a tratamiento, con aumentos paroxisticos de la TA que se acompañan de taquicardia, palpitaciones, sudoración y temblor. Se descarta enfermedad renal, enfermedad vascular renal y ateroesclerosis. Estudios de laboratorio cortisol, aldosterona y renina normal. Electrolitos sodio, potasio y calcio normales ¿Con esta sintomatología, datos de exploración física y exámenes de laboratorio cuál es su diagnóstico?. Sindrome de Conn. Feocromocitoma. Adrenalitis autoinmune. Adenoma hipofisiario. Paciente masculino de 30 años con hipertensión arterial refractaria a tratamiento, con aumentos paroxisticos de la TA que se acompañan de taquicardia, palpitaciones, sudoración y temblor. Se descarta enfermedad renal, enfermedad vascular renal y ateroesclerosis. Estudios de laboratorio cortisol, aldosterona y renina normal. Electrolitos sodio, potasio y calcio normales ¿Qué estudios complementarios solicitaría para confirmar su diagnóstico?. Repetir los electrolitos séricos. Medición de catecolaminas y sus metabolitos (ácido vanimandélico) en orina. Medición de auto anticuerpos. Medición de hormonas hipofisiarias. A 1 year old male with a right abdominal mass is admitted to the University Hospital for check. up, physical examination reveals a large mass on right flank. CT scan shows a large tumor on retropertoneum, which has large areas of necrosis and hemorrhage as well as multiple calcifications are noted, high levels of specific neuron enolase in serum are found and vani-mandelic acid is detected in urine. Which of the following neoplasms is the most likely diagnosis?. Nephroblastoma. Osteosarcoma. Neuroblastoma. Lymphoma. Hepatoblastoma. Masculino de 4 años. Es llevado al servicio de Pediatría por presentar cefalea intensa. A la exploración física presenta hipertensión arterial. Los estudios de laboratorio reportan elevación de las catecolaminas. De acuerdo al cuadro clínico anterior indique el diagnóstico más probable: Nefroblastoma. Neuroblastoma. Hepatoblastoma. Retinoblastoma. Femenina de 32 años. Acude al servicio de medicina interna por presentar aumento de peso. A la exploración física presenta obesidad central, fascies de luna llena y jiba de búfalo. Los estudios de laboratorio reportan cortisol elevado y ACTH baja. De acuerdo al cuadro clínico anterior indique el diagnóstico más probable: Adenoma hipofisiario. Tumor de pulmón de células pequeñas redondas y azules. Tumor suprarrenal. Hombre de 20 años de edad, estudiante de medicina en la UANL, un mes de evolución de edema en cara y miembros inferiores, astenia, adinamia y disminución leve del volumen urinario. Consultó al HU dónde encuentran TA 140/100 FC 82 x Min, FR 18 x min, edema bipalpebral. Al ingreso encuentran paciente Hb 12 g/dl, leucocitos 10.000, Creatinina 3,8 mg/dL, BUN: 50 mg/d, proteinuria 1.000 mg/24 horas. Complemento bajo. Con los datos clínicos y de laboratorio encontrados en este paciente cual seria su diagnostico a considerar de los abajo enlistados: Hipertensión en fase acelerada. Sindrome nefrótico. Sindrome nefrítico. Netropatia en fase rápidamente progresiva. Microangiopatia urémica. Hombre de 20 años de edad, estudiante de medicina en la UANL, un mes de evolución de edema en cara y miembros inferiores, astenia, adinamia y disminución leve del volumen urinario. Consultó al HU dónde encuentran TA 140/100 FC 82 x Min, FR 18 x min, edema bipalpebral. Al ingreso encuentran paciente Hb 12 g/dl, leucocitos 10.000, Creatinina 3,8 mg/dL, BUN: 50 mg/d, proteinuria 1.000 mg/24 horas. Complemento bajo. Se somete a una biopsia renal y el examen microscópico descubre proiferación endotelial, incremento del mesangio y abundantes neutrófilos. En IF hay depósitos de IgG y C3 granular en MB. Con esos datos usted espera que en la Microscopia Electronica se le reporten: Depósitos electrodensos subendoteliales irregulares. Depositos electrodensos globulares epimembranosos. Depositos electrodensos en "joroba" subepiteliales. Depósitos electrodensos mesangiales difusos. Sin evidenca de depositos electrodensos. Hombre de 20 años de edad, estudiante de medicina en la UANL, un mes de evolución de edema en cara y miembros inferiores, astenia, adinamia y disminución leve del volumen urinario. Consultó al HU dónde encuentran TA 140/100 FC 82 x Min, FR 18 x min, edema bipalpebral. Al ingreso encuentran paciente Hb 12 g/dl, leucocitos 10.000, Creatinina 3,8 mg/dL, BUN: 50 mg/d, proteinuria 1.000 mg/24 horas. Complemento bajo. El diagnóstico más consistente con los datos a la mano en este paciente es: Glomeruionefritis postinfecciosa. Glomnerulonefritis con semlunas. Glomeruionnetritis antimembrana basal. Gomerulonefts paucinmune. Glomeruionetritis con asas de alambre. Paciente pediátrico de 5 años de edad es llevado a consultar por presentar edema en extremidades y en cara de 4 meses de evolución, esta pálido e inapetente, en los exámenes de laboratorio se observa Hb 9.5 grs. Creatinina de 0,9 mg/dL, BUN 18 mg/dL y proteínas en orina de 24. De 7.8 grs/lt. No se documentaron otras anomalías Su primera indicación médica en este paciente será: ECO-Dopler para flojo renal. TAC de niones para valorar obstrucciones. Actividad del complemento en suero. Biopsia renal por punción guiada por US. Iniciar tratamiento a base de esteroides. Paciente pediátrico de 5 años de edad es llevado a consultar por presentar edema en extremidades y en cara de 4 meses de evolución, esta pálido e inapetente, en los exámenes de laboratorio se observa Hb 9.5 grs. Creatinina de 0,9 mg/dL, BUN 18 mg/dL y proteínas en orina de 24. De 7.8 grs/lt. No se documentaron otras anomalías. En vista de los datos clínicos es mas probable que el mecanismo fisiopatogénico de la enfermedad observada por este paciente alteró de forma inicial: El epitelio tubular proximal. La capa parietal de Bowman. El endotelio capitar glomerular. Los procesos podálicos del epitelio. La formación de la matriz mesangial. Producto masculino de 38 semanas de gestación. Los estudios de imagen se observan datos de hipoplasia pulmonar así como riñones aumentados de tamaño, secundarios a la presencia de múltiples quistes además de fibrosis hepática. Se le realiza biopsia para estudio histopatológico de riñón que reporta quistes originados en los túbulos colectores sin daño en los glomérulos. Con estos hallazgos el diagnóstico más probable en este paciente es: Displasia Renal. Riñón poliquístico del adulto. Cambios renales adquiridos. Riñón poliquístico del recién nacido. Masculino de 49 años. Acude a consulta por presentar infecciones urinarias de repetición. En la mayoría de estas ocasiones los cultivos resultaron positivos para Proteus mirabilis. Los estudios de imagen muestran una masa de 4 cm localizada en el polo superior del riñon Derecho. Se realiza nefrectomía, el patólogo reporta abundantes macrófagos de aspecto espumoso y granulomas. De acuerdo al cuadro clínico anterior indique el diagnóstico más probable: Carcinoma Renal de células claras. Oncocitoma. Pielonefritis aguda. Pielonefrits crónica xantogranulomatosa. Angiolipoma. Masculino de 58 años. Acude a consulta de urología por presentar hematuria de 2 días de evolución. Los estudios de imagen muestran un tumor de 2 cm, mal definido, con bordes infiltrantes, necrótico, originado en la pelvis renal. El resto del parénquima renal no presenta alteraciones macroscópicas. De acuerdo al cuadro clínico anterior indique el diagnóstico más probable: Carcinoma Papilar. Carcinoma de Células Claras. Carcinoma Cromófobo. Carcinoma Urotelial. Masculino de 68 años. Acude a consulta de Urgencias por presentar dolor vertebral secundario a fractura patológica. Los estudios de laboratorio muestran un APE 50 ng/ml. Los estudios de imagen muestran un crecimiento del lóbulo posterior prostático. De acuerdo al cuadro clínico anterior indique el diagnóstico más probable: Hiperplasia glandular y nodular. Prostatitis aguda. Prostatitis crónica. Adenocarcinoma de próstata. Prostatitis crónica granulomatosa. Masculino de 70 años de edad. Acude al servicio de urgencias por presenta fractura patológica del cuarta vertebra dorsal. Al interrogatorio intencionado refiere historia crónica de obstrucción urinaria disuria e hidronefrosis. Los estudios de laboratorio reportan un antígeno prostático especifico (APE) de 30 ng/dl (rango normal de 0-4 ng/dl) Se decide realizar estudio de imagen el cual reporta un crecimiento irregular de la próstata. En base al cuadro clínico anterior, señale la región anatómica de la próstata donde esperaría encontrar con mayor frecuencia esta patología: Zona centra. Zona periuretral. Lóbulo posterior. Lóbulos laterales. Masculino de 70 años de edad. Acude al servicio de urgencias por presenta fractura patológica del cuarta vertebra dorsal. Al interrogatorio intencionado refiere historia crónica de obstrucción urinaria disuria e hidronefrosis. Los estudios de laboratorio reportan un antígeno prostático especifico (APE) de 30 ng/dl (rango normal de 0-4 ng/dl) Se decide realizar estudio de imagen el cual reporta un crecimiento irregular de la próstata. En base al cuadro clínico anterior el daño que usted esperaría observar, en los estudios de imagen de la cuarta vertebra dorsal, correspondería a: Lesión osteoclástica. Lesión osteoblástica. Lesión osificante. Lesión condroblástica. Masculino de 30 años. Acude al servicio de urología por presentar aumento de volumen del testículo derecho. A la exploración física presenta un testículo aumentado de tamaño no doloroso. Se realiza estudio de imagen que reporta la presencia de un tumor único, heterogéneo, con necrosis, de bordes mal definidos, que infiltra el tejido adyacente. Se realizan estudios de laboratorio que reportan elevación de la alfa feto proteína (AFP), del PLAP, de la fracción beta de la hormona gonadotrofina coriónica (b-HCG) así como de la LDH. Se programa al paciente para la resección quirúrgica. En base a los hallazgos de laboratorio, edad del paciente, y cuadro clínico cual sería el diagnóstico más probable: Tumor de senos endodérmicos. Coriocarcinoma. Carcinoma Embrionario. Seminoma clásico. Tumor germinal mixto. Masculino de 70 años de edad. Que acude al servicio de urología por presentar un aumento de volumen de testículo derecho. Los estudios de imagen reportan la presencia de un tumor único, homogéneo que involucra todo el parenquimático testicular. Los estudios de laboratorio reportan marcadores como PLAP y LDH negativos. Se realiza la resección quirúrgica del tumor, se envía a estudio histopatológico que reporta la presencia de un tumor caracterizado en su totalidad por túbulos seminiferos atróficos con disminución de la espermatogénesis con presencia por fuera de estos de células de tamaño variable con citoplasma escaso, núcleo hipercromático y con actividad mitótica anormal frecuente dispuestas en forma de monocapas entre estos. En este paciente cual sería el diagnóstico más probable en base al cuadro clínico y la morfología exhibida por el tumor testicular antes descrito: Seminoma clásico. Linfoma. Seminoma espermatocítico. Tumor germinal mixto. Tumor de senos endodérmicos. Masculino de 70 años de edad. Que acude al servicio de urología por presentar un aumento de volumen de testículo derecho. Los estudios de imagen reportan la presencia de un tumor único, homogéneo que involucra todo el parenquimático testicular. Los estudios de laboratorio reportan marcadores como PLAP y LDH negativos. Se realiza la resección quirúrgica del tumor, se envía a estudio histopatológico que reporta la presencia de un tumor caracterizado en su totalidad por túbulos seminiferos atróficos con disminución de la espermatogénesis con presencia por fuera de estos de células de tamaño variable con citoplasma escaso, núcleo hipercromático y con actividad mitótica anormal frecuente dispuestas en forma de monocapas entre estos. En base al diagnóstico, ¿cuál de los siguientes marcadores de inmunohistoquimica le ayudaría a confirmar este diagnóstico?. Alfa-fetoproteína. Lactógeno placentario (PLAP). Antígeno carcinoembrionario. CD-20. CD-117. Femenina de 48 años. Acude a servicio de Ginecología por dispareunia. A la exploración física presenta tumor de 2 cm localizado en el exocérvix. Histologicamente está compuesto por células epiteliales malignas con queratinización. Indique el tipo viral asociado al desarrollo de esta Patología. Herpes virus. Virus del Epstein Barr. Virus del papiloma humano. Citomegalovirus. Masculino de 40 años. Diagnóstico de Cáncer de Testículo. El crecimiento acelerado de las células neoplásicas se produce por el Efecto Warbug. El cual consiste en: Glucolisis aeróbica. Oncometabolismo. Glucolisis anaeróbica. Evasión de la apoptosis. Disminución de ATP. Mujer de 30 años con aumento de perímetro abdominal y malestar pélvico de 5 meses de evolución. Los estudios de imágenes revelan una masa que reemplaza el ovario izquierdo. Se extrae un tumor multilocular lleno de líquido espeso y viscoso al corte presenta espacios quístico revestidos de epitelio mucinoso sin evidencia de atipia. No hay estructuras papilares ni invasión estromal. ¿Cuál de los siguientes es el diagnóstico patológico apropiado?. Adenoma endometrioide de ovario. Tumor de células de la granulosa. Cistadenocarcinoma mucinoso. Cistadenoma mucinoso. Cistadenoma seroso. Mujer de 36 años para con diagnóstico de citología cervicovaginal anormal. Se le realiza biopsia por colposcopia, encontrando células escamosas con pérdida de maduración que ocupan todo el espesor del epitelio, pero respetando la membrana basal. ¿Cuál es el diagnóstico adecuado?. Lesión intraepitelial escamosa de bajo grado. Lesión intraepitelial escamosa de alto grado. Carcinoma micro invasor. Mujer de 35 años, con sangrado vaginal. El examen pélvico revela sangre roja brillante en el canal endocervical. Se identifica una masa exófitica ulcerada en el lado izquierdo del cuello uterino. No hay evidencia de directa extensión del tumor al parametrio. La linfa pélvica los ganglios están ligeramente agrandados, mencione cual es el factor de riesgo aislado más importante para el desarrollo del carcinoma espinocelular del cérvix uterino. Menarquia temprana. Menopausia tardía. Infección por virus del papiloma humano. Multiparidad. Mujer nulípara de 50 años con antecedentes de diabetes. Presenta sangrado menstrual mas abundante en los últimos dos ciclos menstruales, y actualmente presenta manchado en todo el ciclo. l ciclo. En la exploración se observa obesidad(IMC = 32 kg / m2) y la presión arterial es de 160/100 mm Hg. Se le realiza ECO pélvico y se observa un endometrio engrosado además de una masa polipoide en el fondo uterino. Se le realiza histerectomía, el patólogo reporta adenocarcinoma endometrial clásico ¿Cuál de los siguientes representa el precursor con más riesgo de desarrollar el carcinoma de endometrio?. Adenomiosis. Hiperplasia simple sin atipia. Endometriosis. Hiperplasia compleja con atipia. Leiomioma. Mujer obesa de 45 años con obesidad (IMC = 32 kg / m2) con antecedentesde diabetes e hipertensión mal controlada se queja de aumento del flujo sanguíneo menstrual de 3 meses de duración. La biopsia endometrial muestra hiperplasia simple del endometrio. ¿Cuál de los siguientes muy probablemente explica la patogenia de la hiperplasia de endometrio en este paciente?. Exceso de estimulación estrogénica. Exposición a agentes progestacionales exógenos. Historia de endometritis crónica. Historia de uso de anticonceptivos orales. Exposición prenatal al dietilestilbestrol. Mujer de 33 años con antecedentes de menorragia con una historia de 6 meses de fatiga creciente. Los exámenes muestras anemia microcítica hipocrómica (hemoglobina = 8 g / dl). El examen pélvico bimanual revela un útero agrandado conmasas múltiples e irregulares. Se realiza una histerectomía y un tumor firme de aspecto arremolinado en el cuerpo uterino de 4cm de diámetro además de 5 más dispuestos en el endometrio y 3 subserosos, señale el diagnostico más probable: Adenomiosis. Hiperplasia simple sin atipia. Endometriosis. Hiperplasia compleja con atipia. Leiomioma. Una mujer de 52 años se presenta con malestar pélvico crónico. La tomografía computarizada de la pelvis muestra una masa uterina con zonas de menor densidad. Se realiza una histerectomía. En un examen general, la masa es blanda con áreas de necrosis irregular y zonas de aspecto mixoide, con bordes que se extienden hacia el miometrio. El examen histológico muestra grandes zonas de necrosis y áreas mixoides rodeadas por un borde de células fusiformes desorganizadas que presentan numerosas mitosis. La tinción inmunohistoquímica para la actina del músculo liso es positiva. ¿Cuál de los siguientes es el diagnóstico más probable?. Adenomiosis. Leiomioma. Adenocarcinoma de endometrio. Leiomiosarcoma. Tumor mixto mesodérmico maligno (Mülleriano). Mujer de 60 años postmenopáusica que presenta sangrado uterino anormal (postmenopáusico), El ECO pélvico muestra un útero con engrosamiento de la línea endometrial y un tumor solido en el ovario derecho. La biopsia endometrial muestra hiperplasia simple del endometrio, usted sospecha hiperestrogenismo, señale cuál de los tumores ováricos pudiera ser el origen. ¿Cuál de los siguientes tumores ováricos pudiera explicar el hiperestrogenismo en esta paciente?. Cistadenoma seroso. Cistadenocarcinoma mucinoso. Tumor de células de la granulosa. Tumor de Brenner. Femenina de 35 años sin antecedentes relevantes, que ingresa a urgencias con dificultad respiratoria, a la exploración se encuentra abdomen globoso, los estudios de imagen muestran derrame pleural derecho, ascitis y un tumor ovárico sólido de 4cm, se coloca sonda en tórax y drena liquido seroso, se realiza cirugía exploradora y se reseca el tumor ovárico el cual es de superficie lisa y al corte es de aspecto fibroso, el patólogo reporta fibroma ovárico, menciona el nombre del síndrome que presenta esta paciente. Síndrome de feminización. Síndrome de Meigs. Síndrome de Nelson. Síndrome de Stein Leventhal. Mujer de 40 años con una historia de 5 años de dismenorrea. El examen físico y los estudios endocrinos son normales. Se realiza una histerectomía. El examen histológico de la pared uterina revela áreas de Adenomiosis extensa. ¿Cuál de las siguientes opciones describe mejor los cambios histológicos de esta patología?. Neoplasia benigna de células epiteliales glandulares. Desplazamiento de glándulas endometriales y estroma en la pared uterina. Neoplasia intraepitelial endometrial. Hiperplasia de trofoblasto como secuela de aborto incompleto. Lesión uterina premaligna compuesta tejido endometrial atípico. Mujer de 50 años nulípara, con antecedentes familiares de cáncer de mama, con 6 meses de evolución con aumento de la circunferencia del abdomen. Tras un interrogatorio minucioso, refiere 1 año de dolor vago en la parte baja del abdomen. El examen pélvico bimanual revela una masa anexial derecha de 10 cm. La ecografía muestra liquido de ascitis y confirma el tumor ovárico. La citología por aspiración de la ascitis reporta estructuras papilares malignas con cuerpos de psamoma. ¿Qué gen mutado es más probable que esté asociado a la neoplasia del ovario de esta paciente?. KRAS (oncogen familia RAS). VHL (gen supresor Von Hippel Lindau). BRCA1 (gen supresor ). WT-1 (gen supresor del Tumor de Wilms). Femenina de 30 años en estudios de esterilidad, con antecedente de ciclos menstruales irregulares, a la exploración presenta exceso de peso e hirsutismo, las hormonas presentan desequilibrio estrogénico, se realiza ecografía y se encuentra ambos ovarios con engrosamiento de la pared y múltiples quistes de aspecto folicular de diversos tamaños, no se observan cuerpos blancos. Mencione el diagnostico más probable. Quiste folicular. Síndrome de ovario poliquístico. Quiste del cuerpo lúteo hemorrágico. Mujer de 60 años con sangrado uterino anormal. El examen ginecológico revela un agrandamiento del útero. La pieza de histerectomía muestra un gran tumor polipoide que infiltra el endometrio y el miometrio. El estudio histológico muestra una proliferación de células de aspecto epitelial malignas que se disponen formando glándulas que infiltran el estroma además de proliferación de células fuso celulares de aspecto sarcomatoso algunas células con estriaciones alargadas de aspecto de renacuajo, otras con atipia y pleomorfismo rodeadas de matriz osteoide inmaduro otras en lagunas de aspecto cartilaginoso maligno, todas muestra abundantes mitosis y zonas de necrosis. ¿Cuál sería es el diagnóstico apropiado?. Adenomiosis. Leiomioma. Adenocarcinoma de endometrio. Leiomiosarcoma. Tumor mixto mesodérmico maligno Mülleriano. Mujer de 65 años con obesidad y DM tipo 2, que inicia con sangrado transvaginal por lo cual le realizan ultrasonido el cual muestra una línea endometrial de 1 cm. Se toma biopsia la cual muestra células que se disponen formando estructuras glandulares complejas fusionándose espalda con espalda y con focos de metaplasia escamosa. De acuerdo a su diagnóstico ¿Cuál es la lesión precursora?. Endometrio atrófico. Polipo endometrial. Neoplasia intraepitelial endometrial. Carcinoma intraepitelial endometrial seroso. Femenina de 35 años. Acude al servicio de Ginecología por presentar menorragia y dismenorrea. Se realiza histerectomía y oforectomía bilateral. El reporte histopatológico del ovario derecho describe un quiste de 2 cm de diámetro mayor con abundante material hemorrágico en su interior. El examen microscópico reporta una pared quística con hemorragia, hemosiderina y glándulas endometriales. Indique el diagnóstico más probable: Adenomiosis ovárica. Endometriosis ovárica. Quiste folicular. Carcinoma de células claras. Cuerpo lúteo hemorrágico. Femenina de 48 años. Acude a urgencias por presentar dolor abdominal intenso. Dentro de sus antecedentes refiere diagnóstico de endometriosis. Los estudios de imagen muestran un tumor de 5 cm en el ovario derecho, el cual es sólido, con áreas de necrosis.El reporte de patología menciona que es una lesión maligna compuesta por células epitelilales que se disponen formando glándulas con núcleos pseudoestratrifcados. Los estudios moleculares reportan alteraciones del PTEN . Indique su diagnóstico más probable: Cistadenocarcinoma mucinoso. Cistadenocarcinoma seroso. Carcinoma endometrioide. Tumor de Brenner. Disgerminoma. Masculino de 45 años. Acude al servicio de Urología por presentar aumento de volumen del testículo izquierdo. Los estudios de imagen muestran un tumor de 2 cm, solido heterogeneo. Se realiza una prueba de embarazo la cual resulta positiva. Indique la explicación de este resultado. El paciente presenta un Tumor del saco vitelino. El paciente presenta un Coriocarcinoma. El paciente presenta un Teratoma postpuberal. El paciente presenta un Seminoma. El paciente presenta un Carcinoma embrionario. Femenina de 56 años. Que acude a consulta por presentar sangrado uterino anormal. El ultrasonido reporta una línea endometrial engrosada, irregular, además de un tumor anexial derecho dependiente del ovario, el cual es sólido y quístico. Se decide realizar biopsia endometrial la cual reporta una neoplasia maligna de origen endometrial que forma estructuras glandulares. De acuerdo al cuadro clínico anterior, el resultado de la biopsia de endometrio corresponde a: Hiperplasia de endometrio compleja atípica. Hiperplasia de endometrio compleja sin atipia. Hiperplasia simple sin atipia del endometrio. Hiperplasia simple con atipia del endometrio. Adenocarcinoma del endometrio. Femenina de 56 años. Que acude a consulta por presentar sangrado uterino anormal. El ultrasonido reporta una línea endometrial engrosada, irregular, además de un tumor anexial derecho dependiente del ovario, el cual es sólido y quístico. Se decide realizar biopsia endometrial la cual reporta una neoplasia maligna de origen endometrial que forma estructuras glandulares. En relación al tumor ovárico, señale la variedad histológica más frecuentemente asociada a este cuadro clínico que presenta la paciente: Cistadenoma seroso. Cistadenocarcinoma mucinoso. Carcinoma endometroide. Tumor de Krukenberg. Tumor de células de la granulosa. Una mujer de 60 años se presenta con 6 meses de aumento la circunferencia abdominal. El examen ginecológico revela tumores ováricos bilaterales sólidos. El paciente se somete a una ooforectomía bilateral. El patólogo diagnostica "tumor de Krukenberg". ¿Cuál de las siguientes las pruebas sería útil en el diagnóstico?. Nivel de AFP (Alfa Feto Proteina) en suero. Biopsia de cuello uterino y legrado endometrial. Laparoscopia. Nivel sérico de HGC (hormona Gonadotropina Corionica). Endoscopia de tubo digestivo. Mujer de 17 años con antecedente de prueba de embarazo casera positiva una semana antes y que inicia con sangrado vaginal. En la exploración vaginal se observa descarga de coágulos y restos de tejido con apariencia de uvas pequeñas. Un ultrasonido muestra una cavidad endometrial dilatada pero sin evidencia de un feto. Se realiza legrado endometrial por aspiración muestra el mismo tejido con apariencia de racimos de uva, que miden hasta 5 mm de diámetro. Mencione el diagnóstico más probable. Coriocarcinoma. Mola parcial. Mola completa. Mola invasiva. Mujer de 25 años en control de embarazo. El ECO pélvico muestra en ovario derecho un tumor de 5cm, quístico con un nódulo mural, con áreas de calcificaciones, ¿Cuál es el diagnóstico más probable?. Disgerminoma. Teratoma quístico maduro. Cistadenoma mucinoso. Cistadenocarcinoma seroso. Femenina de 25 años. Acude a consulta por presentar amenorrea de 3 semanas de evolución. Se realizan estudios de laboratorio los cuales reportan una prueba inmunológica positiva de embarazo, además de niveles elevados al triple HGC para la edad gestacional de la paciente. A la exploración física presenta un útero aumentado de tamaño para la edad gestacional. El ultrasonido demuestra la presencia de un producto no viable, además de múltiples vellosidades dilatadas de aspecto quístico y el estudio de patología muestra que estas son grandes edematosas y avasculares rodeadas de un epitelio trofoblástico. De acuerdo al cuadro clínico anterior, señale su posible diagnóstico: Enfermedad trofoblástica benigna (mola hidatiforme completa). Coriocarcinoma. Mola invasora (corio-adenoma destruens). Enfermedad trofoblástica benigna (Mola hidatiforme incompleta). Femenina de 20 años. Acude a consulta de Ginecología por amenorrea de 1 mes de evolución. A la exploración física se observa un útero de mayor tamaño para la edad gestacional. El diagnóstico histopatológico corresponde a una mola hidatidiforme completa. Indique lo que esperaría encontrar en la biopsia. Vellosidades coriónicas grandes con una cavidad quística central y recubiertas por una proliferación extensa de trofoblasto. No se observan tejidos fetales. Vellosidades coriónicas agrandadas focalmente con una hiperplasia trofoblástica focal. Se observan tejidos fetales. Proliferación del sincitotrofoblasto y citotrofoblasto. No se observan vellosidades coriales. Mujer de 53 años con nódulo en mama. El examen físico confirma una masa en la parte cuadrante superior externa de mama izquierda. La mamografía demuestra un nódulo mal definida, de bordes estrellados de 1 cm. La Biopsia por aspiración con aguja fina (BAAF) muestra células epiteliales ductales malignas. Se realiza mastectomía radical modificada, (resección de mama, piel, pezón, fascia profunda, segmento de músculo pectoral y vaciamiento axilar), la pieza quirúrgica revela un tumor irregular, firme. ¿Cuál de los siguientes marcadores tumorales es serían los más útiles para evaluar antes de considerar opciones terapéuticas para este paciente?. AFP (Alfa Feto Proteina). Receptores de estrógeno y Progesterona (RE y RP). Galactosiltransferasa. Hidrolasas ácidas lisosomales. ACE (Antigeno Carcino Embrionario). Mujer nulípara de 35 años con malestar e hinchazón de ambas mamas a la exploración se encuentran ambas con nodulaciones fluctuantes de aspecto quístico, se realiza mamografía y se observa focos de clasificación, quistes y fibrosis en ambas mamas. ¿Cuál sería el diagnóstico más probable?. Carcinoma ductal in situ (Neoplasia intraductal in situ). Cambio fibroquístico (Enfermedad firboquistica, Mastopatia fibroquistica, mastitis fibrosa quistica). Fibroadenoma. Mastitis granulomatosa. Papiloma intraductal. Femenina de 16 años con nódulo en mama. La mamografía muestra un tumor de 3cm lobulado, se reseca y el patólogo observa una neoplasia compuesta de proliferación de células epiteliales revistiendo los ductos rodeadas de una proliferación del estroma, todo es de aspecto benigno, mencione cual es el diagnóstico más probable. Adenoma de la lactancia. Carcinoma de mama. Papiloma intraductal. Fibroadenoma. Mujer de 24 en lactancia presenta fiebre de 38 ° C (101 ° F). El examen físico no muestra flujo vaginal anormal o evidencia de dolor pélvico, pero se observa enrojecimiento en la parte inferior de la piel de la mama izquierda. La paciente deja de amamantar al bebé temporalmente, pero los síntomas persisten, agregándose dolor intenso y edema cutáneo, a la exploración presenta eritema cutáneo y fisuras en piel de areola y pezón ¿Cuál es el diagnóstico más probable?. Mastopatía fibroquística. Ectasia ductal. Mastitis aguda. Mujer de 20 años, quien acude a consejo genético de riesgo de cáncer de mama, ya que su madre, tía y abuela maternas desarrollaron cáncer de mama, ¿Que pruebas podrían ayudar para conocer este riesgo?. C-myc. Receptores de estrógenos. BRCA1. HER2/neu. Rb-1. Mujer de 55 años con tumor en mama derecha, se realiza biopsia por aspiración que muestra células neoplásicas malignas, se realiza mastectomía y se realiza inmunohistoquímica, para HER2/neu, que es positivo. ¿Cuál de los siguientes es el mecanismos genético indicado en esta positividad?. Mutaciones con inserción. Traslocaciones cromosómicas. Amplificación de gen. Poliploidía. Polimorfismos de un solo nucleótido. Hombre de 55 años con masa de 2cm en región pectoral derecha, se realiza biopsia y reporta células malignas de tipo epitelial. La inmunohistoquímica muestra que las células son positivas para HER2/neu y citoqueratina 4 y 11 pero negativas para receptores de estrógenos. ¿Cual sería el diagnostico más probable?. Carcinoma de mama tipo basal. Carcinoma ductal invasivo. Carcinoma lobulillar invasivo. Carcinoma medular. Mujer de 65 años con tumor palpable en mama, el examen muestra un tumor de 5cm suave al tacto, se realiza biopsia por trucut encontrando un tumor formado por la proliferación de células epiteliales en nidos y tubos los cuales se observa que están contenidos en lagos de moco, menciona el diagnóstico histológico más probable. Carcinoma Lobulillar. Carcinoma medular. Carcinoma coloide o mucinoso. Tumor Phyllodes. Hombre de 55 años con masa de 2cm en región pectoral derecha, se realiza biopsia y reporta células malignas de tipo epitelial. La inmunohistoquímica muestra que las células son positivas para HER2/neu y citoqueratina 4 y 11 pero negativas para receptores de estrógenos. Mencione en relación con la paciente anterior cuál es la alteración más importante en el desarrollo del cáncer en esta paciente. Alcoholismo cronico. Mutación del BRCA2. Ginecomastia. Hiperestrogenismo. Mutación del PTEN. Mujer de 60 años con tumor en mama de 3 meses de evolución, la mamografía muestra un tumo de 8cm de diámetro, bien circunscrito bordes redondeados. Se realiza biopsia y el patólogo observa una sobrecrecimiento de células de estroma de aspecto sarcomatoso con abundantes mitosis atípicas que comprime las áreas de aspecto de hendidura revestidas por células epiteliales sin atipia. ¿Cuál sería el diagnostico más probable?. Carcinoma medular. Fibroadenoma. Tumor Phyllodes. Carcinoma metaplásicas. Femenina de 35 años que acude a consultar por nódulo mal definido en glándula mamaria derecha, doloroso en relación al ciclo menstrual. En la exploración física no encuentra masa. El eco de mama muestra alteraciones fibroquísticas. ¿Cuál de los siguientes hallazgos histológicos se considera cambios proliferativos?. Quiste. Fibrosis. Hiperplasia ductal atípica. Metaplasia apócrina. Adenosis. Femenino de 54 años la cual se le detecta tumor en glándula mamaria derecha, se realiza biopsia en la cual reportan, células en anillo de sello que contienen gota de mucina en su interior, núcleos de bajo grado, células discohesivas y no forma túbulos. De acuerdo a su diagnóstico, que esperaría encontrar en estudios con inmunohistoquímica. Positividad de la Cadherina epitelial. Negatividad de la Cadherina epitelial. Positividad del CDH1. Negatividad del CDH1. Poitividad del ATRX. Femenina de 40 años. Acude a consultar al servicio de cirugía general por encontrarse a la autoexploración un nódulo en glándula mamaria izquierda. A la exploración fisica presenta un tumor único, firme no móvil, fijo a planos profundos en el cuadrante superior externo de la glándula mamaria, con retracción del pezon, "piel de naranja" y adenomegalias axilares ipsilaterales. La mamografía muestra una lesión de bordes irregulares de aspecto estrellado o espicuiado con micro-calcificaciones en su espesor. En esta paciente cual sería su diagnóstico más probable de los abajo enlistados: Necrosis Grasa. Papiloma intraductal. Enfermedad Fibroquistica. Fibroadenoma. Adenocarcinoma de glandula mamaria. Femenina de 40 años. Acude a consultar al servicio de cirugía general por encontrarse a la autoexploración un nódulo en glándula mamaria izquierda. A la exploración fisica presenta un tumor único, firme no móvil, fijo a planos profundos en el cuadrante superior externo de la glándula mamaria, con retracción del pezon, "piel de naranja" y adenomegalias axilares ipsilaterales. La mamografía muestra una lesión de bordes irregulares de aspecto estrellado o espicuiado con micro-calcificaciones en su espesor. Señale cual de los abajo enlistados se considera un factor predisponente para desarrollar esta patología: Multiparidad. Edad de inicio de la vida sexual. Menarquia temprana. Múltiples parejas sexuales. Masculino de 5 años con protrusión del globo ocular derecho, la imagen muestra un tumor en los tejidos blandos de la órbita, se realiza biopsia y se observa una neoplasia maligna compuesta de proliferación de células neoplásicas de aspecto alargado unas con aspecto de renacuajo de citoplasma acidófilo intenso con estriaciones y con núcleo hipercromático algunas otras de aspecto muscular primitivas entre las que se observan zonas ce células con escaso citoplasma de aspecto primitivo. Mencione la variedad de rabdomiosarcoma que es más probable que presente este paciente. Rabdomiosarcoma alveolar. Rabdomiosarcoma pleomórfico. Rabdomiosarcoma embrionario. Menor de 16 años con tumor en tejidos blandos de brazo izquierdo, se realiza biopsia observándose una proliferación neoplásica caracterizada por la formación de estructuras de aspecto alveolar las cuales revestidas por células de citoplasma acidófilo denso con núcleos hipercromáticos, atipia, pérdida de cohesión e invasión a los tejidos blandos del brazo se observan mitosis frecuentes y atípicas. Además se encuentran dos ganglios axilares aumentados de tamaño. Señale el diagnóstico más probable. Rabdomiosarcoma pleomórfico. Rabdomiosarcoma embrionario. Rabdomiosarcoma alveolar. Tumor neuroectodérmico primitivo. Neuroblastoma. Femenina de 20 años con antecedente de colecistectomía 3 meses antes. Consulta por aumento de volumen en el área de la herida quirúrgica en la pared abdominal, a la exploración presenta en dicha área un nódulo de 2 cm de bordes mal definidos y firme a la palpación, se reseca y se observa que consiste en una proliferación de fibroblastos y miofibroblastos de aspecto benigno. ¿Cuál sería el diagnostico mas probable?. Tumor desmoide. Fascitis nodular. Enfermedad de Peyronie. Mujer de 68 años se presenta con un bulto en los tejidos blandos del cuello. El examen físico revela un tumor subcutáneo de 3 cm. La biopsia de la masa muestra una neoplasia benigna. Mencione cuál el tumor de los tejidos blandos más común. Fibroma. Liposarcoma. Lipoma. Adenoma pleomórfico. Rabdomiosarcoma. Masculino con perdida de 10 kg en seis meses, se realizan estudios de laboratorio encontrando solo anemia los estudios de imagen revelan una gran nasa retroperitoneal de aspecto adiposo, con necrosis. Mencione el diagnóstico mas probable con estos hallazgos. Fibroma. Liposarcoma. Lipoma. Adenoma pleomórfico. Rabdomiosarcoma. Masculino de 70 años con alteraciones en mano derecha al presentar contractura en flexión del 4º sobre el 5º dedo. Mencione el diagnostico mas probable. Enfermedad de Ledderhose. Fibromatosis de Dupuytren. Enfermedad de Peyronie. Mujer de 40 años con dolor e hinchazón en el codo izquierdo de más de 6 meses. El examen físico revela una masa de tejido blando de 2 cm. La biopsia muestra una proliferación bifásica de células epiteliales cuboidales dispuestas en estructuras de aspecto glandular y otras fusiformes en fascículos ambas con atipia anaplasia y mitosis. ¿Cuál de los siguientes es el diagnóstico más probable?. Liposarcoma. Histiocitoma fibroso maligno. Rabdomiosarcoma. Sarcoma sinovial. Masculino de 60 años con antecedente de radioterapia que desarrolla un tumor en el muslo derecho de crecimiento rápido en los últimos 2 meses alcanzando un tamaño de 20 cm. La imagen revela que nace de los tejidos blandos, se realiza biopsia encontrando que corresponde a una proliferación de células fusiformes con atipia hipercromatismo dispuestas en haces y remolinos de aspecto de remolinos y más de 10 mitosis en 10 campos, además de zonas de necrosis y otras zonas con marcado pleomorfismo con células multinucleadas de aspecto anaplásico. ¿Cuál es el tumor más frecuente secundario a radioterapia?. Liposarcoma. Histiocitoma fibroso maligno. Rabdomiosarcoma. Sarcoma sinovial. 3 years old male is admitted to the Emergency Room because of fever and maiaise; medical exams reveals urinary infection associated to a large mass in the bladder. Which of the following type is the most frequent neoplasm of the bladder in the pediatric age?. Rabdomiosarcoma pleomorfic. Rabdomiosarcoma alveolar. Rabdomioma. Rabdomiosarcoma botrioids. Femenina de 35 años secretaria con dolor en muñeca izquierda, y limitación del movimiento, a la exploración se observa una lesión nodular en tercio distal de la muñeca, la imagen muestra un tumor epifisiario cona reas líticas irregular que recuerdan burbujas de jabón en el interior del radio con presencia de un trazo de fractura. Se realiza biopsia con aguja fina encontrando una proliferación de células fusiformes además de abundantes células gigantes tipo osteoclástico. Según estos datos ¿Cuál sería el diagnostico más probable?. Osteoma osteoide. Encondroma. Tumor de células gigantes. Ganglión. Masculino de 68 años no fumador, con dolor en fémur derecho y columna lumbar, se realiza estudio de imagen encontrando múltiples lesiones óseas de tipo blástico en ambas regiones óseas, con lo anterior ¿Cuál sería el diagnostico mas probable en este paciente?. Osteosarcoma secundario. Osteosarcoma primario. Osteoma osteoide. Tumores metastásicos. Femenina de 35 años con múltiples tumores en falanges proximales de dedos de manos y pies, los cuales son deformantes, los estudios de imagen muestran múltiples lesiones en diáfisis de dichas falanges que ensanchan la médula ósea con calcificaciones de aspecto moteado y signo del anillo hiperdenso, se reseca y se observa que es de aspecto cartilaginoso, además se encuentran múltiples hemangiomas, tumores en ambos ovarios y un tumor en Sistema nervioso central ¿Cuál sería el diagnostico más probable de los tumores óseos?. Osteomas múltiples. Tumor de células gigantes. Osteosarcomas. Encondroma múltiples. Femenina de 35 años con múltiples tumores en falanges proximales de dedos de manos y pies, los cuales son deformantes, los estudios de imagen muestran múltiples lesiones en diáfisis de dichas falanges que ensanchan la médula ósea con calcificaciones de aspecto moteado y signo del anillo hiperdenso, se reseca y se observa que es de aspecto cartilaginoso, además se encuentran múltiples hemangiomas, tumores en ambos ovarios y un tumor en sistema nervioso central. Considerando las lesiones encontradas ¿Qué síndrome es más probable que sea el que presenta la paciente?. Síndrome de Gardner. Síndrome de Cowden. Síndrome de Li Fraumeni. Síndrome de Maffucci. Masculino de 10 años con deformidad en muslo izquierdo, acude a consulta por eritema cutáneo y fiebre, la biometría hemática muestra leucocitosis. Se descartan lesiones cutáneas y cualquier foco de infección. Los estudios de imagen muestran un tumor en la diáfisis que infiltra la medula y destruye el hueso, infiltrando los tejidos blandos, el periostio presenta una reacción laminar de aspecto de “tela de cebolla”, se realiza Biopsia por aspiración y el patólogo observa una proliferación de células pequeñas redondas y azules, las cuales se disponen en nidos, usted sospecha sarcoma de Ewing. Mencione que gen esta relacionado a este tumor. Gen RB. Gen de fusión EWSR1/FL11 y expresión del c-myc. Gen P53. Masculino de 10 años con deformidad en muslo izquierdo, acude a consulta por eritema cutáneo y fiebre, la biometría hemática muestra leucocitosis. Se descartan lesiones cutáneas y cualquier foco de infección. Los estudios de imagen muestran un tumor en la diáfisis que infiltra la medula y destruye el hueso, infiltrando los tejidos blandos, el periostio presenta una reacción laminar de aspecto de “tela de cebolla”, se realiza Biopsia por aspiración y el patólogo observa una proliferación de células pequeñas redondas y azules, las cuales se disponen en nidos, usted sospecha sarcoma de Ewing. ¿Cuál de los siguientes hallazgos es característico del tumor del paciente anterior?. Coloración de Rojo Congo para determinar amiloide. Coloración de Fontana para búsqueda de melanina. Coloración de PAS para determinación de glucógeno en el citoplasma. Masculino de 14 años con aumento de volumen en rodilla derecha, a la palpación es dura, se realiza estudio de imagen y se observa una lesión en la región meta epifisiaria de fémur distal la cual infiltra la medula y destruye el hueso con zonas que infiltran la cortical, levantando el periostio dando la imagen de triangulo de Codman además de infiltración a los tejidos blandos con aspecto de rayos de sol, se realiza amputación con diagnostico de osteosarcoma, señale el gen asociado a esta neoplasia maligna. Gen PTEN. Gen RB. Gen IH2. Masculino de 65 años con seis meses de evolución presentando astenia, adinamia y dolor óseo, los estudios de imagen muestran múltiples lesiones líticas costillas, vertebras y en cráneo de aspecto característico de “sacabocado”, usted sospecha de mieloma múltiple, mencione que estudio realizaría para corroborar su diagnóstico. Determinación de Antígeno prostático especifico. Electroforesis de inmunoglobulinas. Medición de fosfatasa alcalina. Masculino de 16 años con tumor en tibia refiere dolor intenso que se alvia con aspirina o acetaminofén, la imagen revela un tumor óseo en tibia distal que nace de la cortical y mide 1.7cm de diámetro. Se reseca observando que está compuesto de trabéculas óseas interconectadas al azar con una sola capa de osteocitos maduros. ¿Cuál sería el diagnóstico mas probable?. Osteoma osteoide. Osteoblastoma. Osteocondroma. Femenina de 60 años con neumonía por neumococo de dos semanas de evolución presenta cefalea y deterioro neurológico. A la exploración presenta fiebre y rigidez de nuca, se realiza punción de Líquido cefalorraquídeo, usted sospecha una meningitis bacteriana, señale que es lo que espera observar en el líquido cefalorraquídeo en este caso: Liquido turbio con abundantes bacterias, elevación de proteínas, leucocitos y glucosa baja. Liquido de aspecto de agua de roca, abundantes linfocitos, y proteínas elevadas y glucosa normal. Recién nacido producto de cesárea con antecedente de ruptura prematura de membranas, presenta fiebre, irritabilidad, se sospecha meningitis bacteriana, señale el agente causal mas probable: Neumococo. Meningococo. E. coli. Masculino de 40 años que refiere haber sido mordido en la mano por un animal salvaje probablemente un coyote, por lo que usted debe prevenir una infección transmitida por estos animales, así que requiere que el paciente sea inmunizado para prevenir la encefalitis por rabia, en caso de que esta infección se desarrolle en el paciente y fallezca que dato citopático se buscaría en las neuronas para determinar la infección: Cuerpos de Psamoma. Cuerpos de Cowdry. Cuerpos de Negri. Masculino con VIH desarrolla alteraciones de la conciencia, ingresa a urgencias y fallece dos días después en la autopsia se observa hemorragia puntiforme en ambos lóbulos temporales, las laminillas histológicas muestran infiltrado linfoide en el parénquima cerebral, además de abndante hemorragia, además de que se encuentran células gliales con datos inclusiones acidófilas denominadas en ojos de buey. Por lo que diagnostica una encefalitis vírica, señale el agente causal mas probable: Virus del Herpes simple. Virus de la inmunodeficiencia humana. Citomegalovirus. Femenina de 62 años con HTA de 25 años de evolución con perdida súbita del conocimiento, en la imagen se observa aneurismas de Charcot-bouchard, en ramas de la arteria cerebral media. En base a esto donde es mas frecuente la hemorragia parenquimatosa: Lóbulo frontal. Lóbulo temporal. Ganglios basales. Cuerpo calloso. Femenina 85 años que ingreso a urgencias con pérdida de la conciencia, refieren historia de traumatismo en cráneo dos meses antes al caerse de su propia altura en forma accidental y datos de demencia en el último mes, una semana antes se agrega perdida lenta de reflejos hasta terminar con pérdida de conciencia, a la exploración presenta midriasis y anisocoria. Se realiza TAC encontrando una lesión blanca de aspecto biconvexo, señale el diagnostico mas probable. Hematoma subdural. Hemorragia subaracnoidea. Hematoma epidural. Mujer de 55 años con cefalea intensa, que desarrolla rápida perdida de la conciencia, crisis convulsivas y a la exploración presenta rigidez de nuca. El estudio del LCR presenta sangrado franco. Se le diagnostica hemorragia subaracnoidea. Señale la causa más frecuente de hemorragia subaracnoidea no traumática originada por ruptura de un aneurisma sacular. Señale la localización más frecuente de localización de los aneurismas saculares. Arteria cerebral media. Unión de la Arteria Cerebral Anterior y Arteria Comunicante Anterior. Ramas intracraneales de la Arteria Carótida Interna. Hombre de 25 años que inicia con crisis convulsivas, se realizan estudios encontrando en la angiografía cerebral una zona compuesta de vasos ramificados con corto circuito izquierda-derecha. Se realiza resección y el patólogo reporta una lesión compuesta con abundantes vasos sanguíneos dilatados de paredes de grosor variable algunos con una lámina elástica interrumpida y capa intima engrosada y fibrosa, alternando con vasos de pared muy delgada sin fibras elásticas. ¿Qué tinción realizó el patólogo para demostrar la presencia de fibras elásticas en una malformación artería venosa (MAV). PAS. Tricrómico. Tinción de Verhoeff-Van Gieson. Mujer de 38 años con crisis convulsivas, se observa un tumor en lóbulo frontal en la sustancia blanca con calcificaciones, se realiza biopsia transoperatoria el patólogo describe una proliferación de células que se disponen en nidos con una fina red de vasos capilares, las células presentan núcleos redondos rodeados de un halo claro dando el aspecto de “Huevo estrellado”, las cuales presentan mutacion del gen IDH. Mencione el diagnostico más probable: Astrocitoma pilocítico. Meduloblastma. Oligodendroglioma. Niño 3 años con ictericia se realiza una ecografía y reportan un tumor hepático de 4 cm con zonas de necrosis y áreas densas con calcificaciones, se sospecha un Hepatoblastoma, señale el marcador tumoral que mediría en sangre y que aumenta en este tumor en niños. ACE. AFP. CA19-9. Niño de 1 año con control prenatal y sin antecedentes familiares de cáncer en la familia, la madre lo lleva a consulta porque nota una coloración blanquecina en el fondo de ojo, el cual ha notado que se desvía, a la exploración encuentra un tumor en fondo de ojo grisáceo en dependiente de la retina, la imagen muestra que ocupa el 20% del globo ocular, se reseca y el patólogo reporta un Retinoblastoma, según los antecedentes de este paciente señale el tipo Retinoblastoma que tiene el paciente. Retinoblastoma esporádico. Retinoblastoma hereditario. Niño de 5 años con protrusión del globo ocular la imagen muestra un tumor dentro de la órbita originado de los músculos perio culares, se realiza biopsia y el patólogo describe una proliferación maligna compuesta de células de citoplasma acidófilo, con núcleos hipercromáticos de aspecto en renacuajo algunas con estriaciones transversales. Señale el diagnóstico más probable. Osteosarcoma. Rabdomiosarcoma. Retinoblastoma. Neuroblastoma. Meduloblastoma. Lactante de 2 años varón con fiebre e irritabilidad, mal estado general, presenta mucosas pálidas, y abdomen globoso. La biometría hemática muestra anemia, el examen general de orina muestra abundantes leucocitos y eritrocitos, el ECO de vejiga muestra una tumoración polipoide de 5cm de diámetro mayor que ocupa la luz vesical. Se reseca y el patólogo describe una neoplasia maligna de origen sarcomatoso, señale el tipo de sarcoma y la variante histológica más probable. Osteosarcoma. Rabdomiosarcoma clasico. Rabdomiosarcoma botroiodes. Retinoblastoma. Neuroblastoma. Niño de 10 años con lesión eritematosa en pierna derecha, fiebre y dolor a la palpación, la cual es dura no evidenciando lesiones cutáneas ni infecciones en otras áreas, la biometría muestra leucocitosis, la imagen radiológica describe una lesión de aspecto de tela de cebolla usted sospecha una neoplasia maligna, señale el diagnóstico y la alteración genética más probable: Traslocacion 11,22 expresión del n-myc: Sarcoma de Ewing. Deleción 11: sarcoma de Ewing. Mutación del RB: Osteosarcoma. Mutación p53: osteosarcoma. Niño de 6 años con síndrome cerebeloso, la resonancia muestra un tumor en 4º. Ventrículo la imagen muestra un quiste con un nódulo mural, se reseca y el patólogo describe una proliferación de atrocitos bipolares con fibras de Rosenthal y entre estas se observan cuerpos granulares y zonas con micro quistes, según la clasificación de la OMS de los tumores gliales, señale a que tipo corresponde el Astrocitoma pilocítico y su significado. I: Tumores circunscritos, de lento crecimiento. II: Tumores de borde difuso, lento crecimiento y algunos con tendencia a progresar a mayor malignidad. III: Tumores infiltrantes con atipia y anaplasia. IV: Tumores de rápido crecimiento, muchas mitosis, neo formación vascular y necrosis. Niño de 15 años con ataxia y nistagmo, la imagen muestra un tumor en cuarto ventrículo, con calcificaciones, se reseca y el patólogo describe que está formado de células pequeñas redondas y azules que forman seudo rosetas de Hommer-Wright con centro germinal, histológicamente lo describen como clásico, el estudio genético revela mutación con activación del WNT, mencione la variedad histológica esperada y su pronóstico en esta variante genética del tumor. Meduloblastoma Desmoplásico pronostico intermedio. Meduloblastoma clásico buen pronostico. Meduloblastoma anaplásico mal pronostico. Masculino de 25 años que inicia su padecimiento con cefalea y vómito, llega a urgencias con pérdida de la conciencia. Los estudios de imagen muestran en lóbulo temporal derecho una lesión con necrosis central rodeada de un anillo hiperdenso y edema cerebral. Se decide operar observandose en la biopsia una lesión glial con aumento en la celularidad, atipia nuclear, mitosis, hiperplasia endotelial y necrosis con empalizada periférica. Dentro de sus antecedentes refiere diagnóstico previo de astrocitoma grado II Indique ¿Cuál es su diagnóstico y subtipo molecular?. Astrocitoma difuso IDH mutante. Astrocitoma anaplasico IDH mutante. Oligoastrocitoma IDH mutante. Glioblastoma multiforme IDH mutante. Glioblastoma multiforme IDH tipo natural. Masculino de 37 años con tumor cerebral, se realiza biopsia y encuentran en la neoplasia mutacion del IDH y codelecion 1p y 19q. Indique ¿cuál sería la posibilidad diagnóstica qué pensaría primero?. Glioblastoma multiforme. Ependimoma mixopapilar. Astrocitoma anaplasico. Linfoma primario de cerebro. Oligodendroglioma. Masculino de 13 años. Es llevado por sus padres al servicio de traumatología por presentar una lesión pediculada en la rodilla izquierda. Histologicamente la lesión está compuesta por una cubierta de cartílago hialino benigno y un pedículo de hueso maduro. De acuerdo al cuadro clínico anterior indique el sitio donde sería más posible encontrar esta lesión: Diafisis. Epífisis. Metáfisis. Periostio. Masculino de 40 años sin antecedentes médicos de importancia que sufre accidente automovilístico Es trasladado al hospital por pérdida de la conciencia. Llega a urgencias inconsciente. La TAC mostró contenido hemático desde la fosa temporal izquierda hasta la convexidad parietal de aspecto de semiluna. Indique ¿Cuál es su diagnóstico?. Hematoma subdural. Hematoma intraparenquimatoso. Hematoma epidural. Hematoma intraventricular. Hemorragia subaracnoidea. Hombre de 45 años que acude a consultar por presentar lesión en espalda de 1-2cms de diámetro, papulo-nodular y pigmentada que clínicamente presenta una coloración irregular, con tintes de colores marrón oscuro y negro. Las secciones realizadas muestran una proliferación neoplásica de células melanociticas con atipia importante que se disponen en nidos sólidos con un crecimiento vertical, que infiltran la totalidad de la dermis papilar. En base a su diagnóstico de este proceso. ¿Cuál de los siguientes subtipos de dichos procesos neoclásicos considerado por usted anteriormente es el de peor pronóstico y comportamiento biológico más agresivo?. Melanoma maligno de distribución Pagetoide. Melanoma de extensión superficial. Melanoma nodular. Melanoma sacral lentiginoso. Melanoma lentigo maligno. Hombre de 45 años que acude a consultar por presentar lesión en espalda de 1-2cms de diámetro, papulo-nodular y pigmentada que clínicamente presenta una coloración irregular, con tintes de colores marrón oscuro y negro. Las secciones realizadas muestran una proliferación neoplásica de células melanociticas con atipia importante que se disponen en nidos sólidos con un crecimiento vertical, que infiltran la totalidad de la dermis papilar. En este paciente, además de la variedad histológica, ¿Cuál es el indicador pronóstico independiente de sobrevida mas importante y que tiene impacto en la determinación del margen quirúrgico?. Nivel de Clark. Índice de Breslow. Migración pagetoide. La presencia de lesión precursora. El tamaño macroscópico en diámetro de la lesión. Hombre de 45 años que acude a consultar por presentar lesión en espalda de 1-2cms de diámetro, papulo-nodular y pigmentada que clínicamente presenta una coloración irregular, con tintes de colores marrón oscuro y negro. Las secciones realizadas muestran una proliferación neoplásica de células melanociticas con atipia importante que se disponen en nidos sólidos con un crecimiento vertical, que infiltran la totalidad de la dermis papilar. Las secciones histológicas practicadas al tumor en este paciente muestran la presencia de células neoclásicas infiltrando la totalidad de la dermis reticular. ¿A qué nivel de Clark corresponde este tumor?. Nivel de Clark I. Nivel de Clark II. Nivel de Clark III. Nivel de Clark IV. Nivel de Clark V. Masculino de 60 atos sin antecedentes de mportancia que inicia su padecimiento con céfalea intensa y vomito, los estudios de imagen muestran una lesión mal definida hipodensa con necrosis central rodeada de un anillo hiperdenso. Se toma biopsia la cual muestra una lesión con fondo fibrilar, con aumento en la celularidad, atipia nuclear, mitosis, hiperplasia endotelial y necrosis en "pseudoempalzada". ¿Cuál es su diagnóstico?. Astrocitoma piocitico. Astrocitoma anaplasico. Glioblastoma multiforme. Oligodendroglioma. Masculino de 60 atos sin antecedentes de mportancia que inicia su padecimiento con céfalea intensa y vomito, los estudios de imagen muestran una lesión mal definida hipodensa con necrosis central rodeada de un anillo hiperdenso. Se toma biopsia la cual muestra una lesión con fondo fibrilar, con aumento en la celularidad, atipia nuclear, mitosis, hiperplasia endotelial y necrosis en "pseudoempalzada". Este paciente presenta una lesión primaria o de "novo", ¿Cuál es la alteración genética frecuentemente en estos pacientes?. Delección del cromosoma 19q. Ampificación del gen EGFR. Amplificación del c-myc. Mutación del gen del retinoblastoma. Femenina de 45 años con presencia de múltipies lesiones deformantes en región de manos y de pies, caractarizadas radiologicamente por lesiones expansivas de los huesos tubulares cortos de bordes adelgazados y de aspecto cartilaginoso. Ademas la paciente muestra lesión en cara caracterizadas por una neoformación vascular de color rojiza bien circunsonta. Además de presentar alteraciones de la conducta con movimientos atetósicos y ataxicos por lo que se éfectua estudio de T.A.C, en la que se observa lesión parenquimatosa cerebral con nódulo mural en lóbulo temporal. Con los hallazgos clínico-radiológicos encontratios en esta paciente usted piensa que se trata de una paciente con: Sindrome de Meigs. Sindrome de Oller. Síndrome de Maffucci. Sindrome de Becwith-Widemann. Masculino de 12 años el cual presenta deformidad a nivel de la pierna izquierda además de observarse la presencia de una dermatosis caracterizada por edema con aumento local de la temperatura, los estudios de laboratorio muestran aumento de la fosfatasa alcalina, así como aumento de la velocidad de sedimentación giobular, los estados radiológicos muestran la presencia de un tumor diafisiario a nivel de la tibia con adelgazamiento y ruptura de la cortical con presencia de Triángulo de Codman, ademas de encontrarse una reaccion periostica concentrica a nivel del tumor, por lo que a esta paciente se le realiza toma de biopsia y en la que el estudio histopatológico se encuentra una lesión de células pequeñas redondas y obscuras maligna. Con estos hallazgos clinico-radio-patologicos usted piensa que el paciente tiene cual de los siguientes tumores óseos: Sarcoma osteogenico. Condrosarcoma. Sarcoma de Ewing. Osteoclastoma (tumor de céluias gigantes). Masculino de 12 años el cual presenta deformidad a nivel de la pierna izquierda además de observarse la presencia de una dermatosis caracterizada por edema con aumento local de la temperatura, los estudios de laboratorio muestran aumento de la fosfatasa alcalina, así como aumento de la velocidad de sedimentación giobular, los estados radiológicos muestran la presencia de un tumor diafisiario a nivel de la tibia con adelgazamiento y ruptura de la cortical con presencia de Triángulo de Codman, ademas de encontrarse una reaccion periostica concentrica a nivel del tumor, por lo que a esta paciente se le realiza toma de biopsia y en la que el estudio histopatológico se encuentra una lesión de células pequeñas redondas y obscuras maligna. ¿Cuál de los siguientes hallazgos de microscopio electrónica confirmaría el diagnóstico del tumor antes señalado?. Presencia de abundantes mitocondrias. Presencia de vacuolas de glucógeno. Presencia de uniones tipo desmosómico. Presencia de melamosomas. Femenina de 70 años. Acude al servicio de dermatología por presentar una lesión a nivel de la región malar derecha de 2 meses de evolución. A la exploración física es nodular elevada de color banco perlado. Se realiza biopsia excisional que reporta una neoplasia maligna originada de la capa basal caracterizada por células monótonas con escaso citoplasma que se disponen formando nidos sólidos con empalizada périferica la cual corresponde a un Carcinoma Basocelular. De acuerdo al diagnostico, cual seria de los abajo enlistados el factor etiológico más importante. Exposición a arsénico. Exposición a benzopirenos. Exposición a la radiación ultravioleta. Exposición a nitritos. Paciente femenina de 45 años que inicia su padecimiento con odinofigia y malestar general, acude a consutar y se le indica tratamiento sintomático, no antibióticos. Al día siguiente desarrolla fiebre que cede con paracetamol y cefalea frontal intensa, ademas de vómitos precedidos de nauseas en tres ocasiones. Acude a urgencias a donde lega con alteración del estado de la conciencia y rigidez de nuca. Se realiza punción de LCR la cual muestra un aumento en la celularidad con predominio de linfocitos, elevación moderada de proteínas y glucosa normal. Los cutivos resultaron negativos. En esta paciente ¿Cuál es su diagnóstico clínico más probable y agente etiológico relacionado?. Meningitis aguda - Bacterias Gram (+). Meningitis crónica - Hongos. Meningitis aséptica (linfocitica) - Virus. Meningios crónica - Mycobacterias. Meningitis aguda - Bacterias Gram(-). Paciente mujer de 34 años, con historia de 2 meses de evolución de cefaleas recurrentes, las que aumentan en intensidad durante la semana previa al ingreso. Inicia con compromiso de la conciencia subito y mientras es evaluada en el servicio de urgencia presenta paro respiratorio requiriendo intubación. El TAC de craneo evidenció hemorragia subaracnoidea. ¿En base a los sintomas clinicos y radiológicos observados en este paciente cuál es su diagnóstico?. Malformación arterio venosa. Angioma cavernoso. Aneurisma sacular. Microaneurisma de Charcot-Bouchard. Paciente mujer de 34 años, con historia de 2 meses de evolución de cefaleas recurrentes, las que aumentan en intensidad durante la semana previa al ingreso. Inicia con compromiso de la conciencia subito y mientras es evaluada en el servicio de urgencia presenta paro respiratorio requiriendo intubación. El TAC de craneo evidenció hemorragia subaracnoidea. En base a su diagnóstico. ¿Cuál es el sitio de afectación más frecuente?. Arteria cerebral media. Unión de la arteria cerebral anterior y comunicante anterior. Arteria comunicante posterior. Arteria carótida interna. Unión de la arteria cerebral posterior y comunicante posterior. |